RegistrierenRegistrieren   LoginLogin   FAQFAQ    SuchenSuchen   
Photonenzerfall verboten in QED
 
Neue Frage »
Antworten »
    Foren-Übersicht -> Quantenphysik
Autor Nachricht
TomS
Moderator


Anmeldungsdatum: 20.03.2009
Beiträge: 18201

Beitrag TomS Verfasst am: 13. Mai 2013 16:13    Titel: Photonenzerfall verboten in QED Antworten mit Zitat

Es geht um den hypothetischen Zerfall



mit einem unbekannten Teilchen-Antiteilchen-Paar (X).

Man kann sich leicht überlegen, dass dieser Zerfall verboten ist, da Energie- und Impulserhaltung nicht gleichzeitig gelten können, es sei denn: m(X) = 0 und p(X) parallel zum ursprüngliche Photonimpuls.

D.h. es bleibt die Möglichkeit, dass ein Photon in zwei Photonen zerfällt, die sich (mit jeweils halber Frequenz, Energie und Impuls) exakt in Vorwärtsrichtung bewegen.

Diesen Zerfall beobachtet man in der Natur nicht, d.h. das u.g. Feynmandiagramm muss in der QED exakt Null sein. Wie sieht man ein, dass da so ist? Antisymmetrie des dp-Integranden im Fermion-Loop und daher Null?



162px-Triangle_diagram_svg.png
 Beschreibung:
 Dateigröße:  2.66 KB
 Angeschaut:  1503 mal

162px-Triangle_diagram_svg.png



_________________
Niels Bohr brainwashed a whole generation of theorists into thinking that the job (interpreting quantum theory) was done 50 years ago.
adaddsa
Gast





Beitrag adaddsa Verfasst am: 13. Mai 2013 17:58    Titel: Antworten mit Zitat

Vielleicht wäre für dich interessant:
"General properties of the decay amplitudes for massless particles
G. Fiore, G. Modanese"
_Reggid
Gast





Beitrag _Reggid Verfasst am: 13. Mai 2013 18:46    Titel: Antworten mit Zitat

darf ich nochmal nachfragen, warum du weißt, dass dieser prozess nicht möglich ist? schließlich ist die photon-photon-streuung (also mit vier externen photonen) über einen fermion-loop ja auch möglich, wenn auch natürlich stark unterdrückt. dieses diagramm könnte ich ja auch für ein einlaufendes und drei auslaufende photonen verwenden, was ja für masselose teilchen mit enerige- und impulserhaltung vereinbar ist. woher kann man unterscheiden, dass der prozess nicht erlaubt, der prozess schon erlaubt ist? ich habe außerdem immer gelernt, dass die photonenzahl sowieso nicht erhalten ist, bzw. nicht genau defniert ist.

wenn du aber meinst, dass dieses diagramm mit drei photonen sicher 0 sein muss, dann wäre mir zuerst noch paritätserhaltung in der QED als argument eingefallen, aber eigentlich sollte das bei dem prozess auch kein problem machen, oder? photon hat parität (-1), d.h. mit einem relativen drehimpuls von 1 für die auslaufenden teilchen sollte das mit parität und auch drehimpulserhaltung (spin der auslaufenden photonen anitparallel) in ordnung gehen.[/latex]
_Reggid
Gast





Beitrag _Reggid Verfasst am: 13. Mai 2013 19:01    Titel: Antworten mit Zitat

kurze google suche hat allgemein "Furry's theorem" ergeben. in diesem speziellen fall sind es eigentlich zwei diagramme (ausgehende photon linien "kreuzen" wegen ununterscheidbarkeit), die sich canceln.
adaddsa
Gast





Beitrag adaddsa Verfasst am: 13. Mai 2013 19:03    Titel: Antworten mit Zitat

_Reggid hat Folgendes geschrieben:

wenn du aber meinst, dass dieses diagramm mit drei photonen sicher 0 sein muss, dann wäre mir zuerst noch paritätserhaltung in der QED als argument eingefallen, aber eigentlich sollte das bei dem prozess auch kein problem machen, oder? photon hat parität (-1), d.h. mit einem relativen drehimpuls von 1 für die auslaufenden teilchen sollte das mit parität und auch drehimpulserhaltung (spin der auslaufenden photonen anitparallel) in ordnung gehen.[/latex]
Das wird doch gerade mit C-Parität nicht passen, wegen (-1)^2
TomS
Moderator


Anmeldungsdatum: 20.03.2009
Beiträge: 18201

Beitrag TomS Verfasst am: 13. Mai 2013 19:59    Titel: Antworten mit Zitat

_Reggid hat Folgendes geschrieben:
kurze google suche hat allgemein "Furry's theorem" ergeben. in diesem speziellen fall sind es eigentlich zwei diagramme (ausgehende photon linien "kreuzen" wegen ununterscheidbarkeit), die sich canceln.

Perfekt, das hatte ich mal gelesen und wieder vergessen.

Man kann sowohl mit der Parität angewandt auf die Matrixelemente, als auch explizit mittels Berechnung der Diagramme argumentieren. Danke!

_________________
Niels Bohr brainwashed a whole generation of theorists into thinking that the job (interpreting quantum theory) was done 50 years ago.
Neue Frage »
Antworten »
    Foren-Übersicht -> Quantenphysik